LSAT and Law School Admissions Forum

Get expert LSAT preparation and law school admissions advice from PowerScore Test Preparation.

 Administrator
PowerScore Staff
  • PowerScore Staff
  • Posts: 8916
  • Joined: Feb 02, 2011
|
#31774
Please post below with any questions!
 jlam061695
  • Posts: 62
  • Joined: Sep 17, 2016
|
#32370
I did pick E for this one, but can someone provide a thorough explanation on why it's correct? I got E through the process of elimination, since I thought all of the other choices did not have to be true (well they had to not have been true, obviously)
 Kristina Moen
PowerScore Staff
  • PowerScore Staff
  • Posts: 230
  • Joined: Nov 17, 2016
|
#32391
Hi jl,

I'm glad you're asking about a question that you got correct. Many students gloss over questions that they got correct when they could learn a great deal from them (and identify their strengths!).

Since this a Must Be True, the four incorrect answers are all things that we can't infer - meaning, they could be true, could be false, or might even have to be false. The correct answer is something that absolutely, unequivocally must be true based on the stimulus.

So the premise tells us that there are multiple reasons why global warming has contributed to a rise in sea level - ice melts (turning into water) and when water is heated, the volume increases. We're also told that the rise has been lessened by artificial reservoirs.

So if you were to diagram this, you could write (where increased sea level is to the right, normally I'd write a vertical arrow but that's harder to do on the computer!):
Ice melts ->
Water heats ->
Reservoirs <-

Answer choice (E) is something that must be true. We can't determine how much water melted off the ice based on the sea level rise alone, since there are other things that affect the rise in sea level!

Is there another answer choice you would like to discuss?
 jlam061695
  • Posts: 62
  • Joined: Sep 17, 2016
|
#32400
Oh that makes sense now! E was the only answer choice I was hesitant about!
User avatar
 relona
  • Posts: 24
  • Joined: Jul 23, 2021
|
#91872
Can someone please explain why is B incorrect?
 Adam Tyson
PowerScore Staff
  • PowerScore Staff
  • Posts: 5153
  • Joined: Apr 14, 2011
|
#91883
Sure thing, relona! We cannot infer answer B (it is not something that "can most reasonably be concluded") because we have no information at all about what was happening before the reservoirs were built. Maybe rises in sea level were easy to explain back then, whether they were due to melting of glaciers, or warming of water, or something else? The stimulus says absolutely nothing about how easy or difficult it was to explain before or about how easy it is now. That's just a whole new idea that is not addressed by the facts given in the stimulus, and a requirement of new information is fatal to a Must Be True/Most Strongly Supported answer choice.

Get the most out of your LSAT Prep Plus subscription.

Analyze and track your performance with our Testing and Analytics Package.